You are on page 1of 87

ICSE Class 10 Physics Previous Year Question Paper

2013
PHYSICS
Science Paper -1
Time: 1½hrs. Maximum Marks: 80

General Instruction
1. Answer to this Paper must be written on the paper provided separately.
2. You will not be allowed to write during the first 15 minutes. This time is to be spent
in
3. reading the Question Paper.
4. The time given at the head of this Paper is the time allowed for writing the answers.
5. Section I is compulsory. Attempt any four questions from Section II.
6. The intended marks for questions or parts of questions are given in brackets [ ].

Section - I
Question 1 [10]
(a) Given any two effects of a force on a non-rigid body.
(b) One end of a spring is kept fixed while the order end is stretched by a force as
shown in the diagram.

(i) Copy the diagram and mark on it the direction of the restoring force.
(ii) Name one instrument which works on the above principle.
(c)
(i) Where is the centre of gravity of uniform ring situated?
(ii) 'The position of the centre of gravity of a body remains unchanged even when
the body is deformed.' State whether the statement is true or false.
(d) A force is applied on a body of mass 20 kg moving with a velocity of 40 ms-1. The
body attains a velocity of 50 ms-1 in 2 seconds. Calculate the work done by the body.
(e) A type of single pulley is very often used as a machine even though it does not give
any in mechanical advantage.
(i) Name the type of pulley used.
(ii) For what purpose is such a pulley used?
Question 2 [10]
(a)
(i) In what way does an 'Ideal machine' differ from a 'Practical machine'?
(ii) Can a simple machine act as a force multiplier and a speed multiplier at the same
time?
(b) A girl of mass 35 kg climbs up from the first floor of a building at a height 4 m above
the ground to the third floor at a height 12 m above the ground. What will be
increase in her gravitational potential energy? (g = 10 ms-2).
(c) Which class of lever found in the human body is being used by a boy-

(i) When he holds a load on the palm of his hand.


(ii) When he raises the weight of his body on his toes?

(d) A ray of light is moving from a rarer medium to denser medium and strikes a plane
mirror placed at 90o to the direction of the ray as shown in the diagram.

(i) Copy the diagram and mark arrows to show the path of the ray of light after it is
reflected from the mirror.
(ii) Name the principle you have used to mark the arrows to show the direction of
the ray.
(e)
(i) The refractive index of glass with respect to air is 1.5. What is the value of the
refractive index of air with respect to glass?
(ii) A ray of light is incident as a normal ray on the surface of separation of two
different mediums. What is the value of the angle of incidence in this case?
Question 3 [10]
(a) A bucket kept under a running tap is getting filled with water. A person sitting at a
distance is able to get an idea when the bucket is about to be filled.
(i) What change takes place in the sound to give this idea?
(ii) What causes the change in the sound?

(b) A sound made on the surface of a lake takes 3 s to reach a boatman. How much time
will it take to reach a diver inside the water at the same depth?
Velocity of sound in air = 330 ms-1
Velocity of sound in water = 1450 mx-1

(c) Calculate the equivalent resistance between the points A and B for the following
combination of resistors:

(d) You have been provided with a solenoid AB.

(i) What is the polarity at end A?


(ii) Give one advantage of an electromagnet over a permanent magnet.
(e)
(i) Name the device used to protect the electric circuits from overloading and short
circuits.
(ii) On what effect of electricity does the above device work?
Question 4 [10]
(a) Define the term 'Heat capacity' and state its S.I. unit.
(b) What is meant by Global warming?
(c) How much heat energy is released when 5 g of water at 20 oC changes to ice at 0 oC?
[Specific heat capacity of water = 4.2 J g-1 oC-1
Specific latent heat of fusion of ice = 336 J g-1]
(d) Which of the radioactive radiations-?
(i) can cause severe genetical disorders.
(ii) are deflected by an electric field?
(e) A radioactive nucleus undergoes a series of decays according to the sequence
  
X  X1  X2  X3.
If the mass number and atomic number of X3 are 172 and 69 respectively, hat is the mass
number and atomic number of X?

Section - II
Question 5 [10]
(a)
(i) With reference to their direction of action, how does a centripetal force differ
from a centrifugal force?
(ii) State the Principle of conservation of energy.
(iii) Name the form of energy which a body may possess even when it is not in
motion.
(b) A coolie is pushing a box weighing 1500 N up an inclined plane 7.5 m long on to a
platform, 2.5 m above the ground.
(i) Calculate the mechanical advantage of the inclined plane.
(ii) Calculate the effort applied by the collie.
(iii) In a actual practice, the collie needs to apply more effort than what is calculated.
Give one reason why you think the coolie needs to apply more effort.
(c) A block and tackle system of pulleys has a velocity ratio 4.
(i) Draw a labelled diagram of the system indicating clearly the points of application
and directions of load and effort.
(ii) What is the value of the mechanical advantage of the given pulley system if is an
ideal pulley system?
Question 6 [10]
(a) Name the radiations:
(i) That are used for photography at night.
(ii) Used for detection of fracture in bones.
o o
(iii) Whose wavelength range is from 100 A to 4000 A (or 10 nm to 400 nm).
(b)
(i) Can the absolute refractive index of a medium be less than one?
(ii) A coin placed at the bottom of a beaker appears to be raised by 4.0 cm.
If the refractive index of eater is 4/3, find the depth of the water in the beaker.
(c) An object AB is place between 2F1 and F1 on the principal axis of a convex lens as
shown in the diagram:

Copy the diagram and using three rays starting from point A, obtain the image of the object
formed by the lens.

Question 7 [10]
(a)
(i) What is the principle on which SONAR is based?
(ii) An observer stands at a certain distance away from a cliff and produces a loud
sound. He hears the echo of the sound after 1.8 s. Calculate the distance between
the cliff and the observer if the velocity of sound in air is 340 ms-1.
(b) A vibrating tuning fork is place over the mouth of a burette filled with water. The tap
of the burette is opened and the water level gradually starts falling. It is found that
the sound from the tuning fork becomes very loud for a particular length of the
water column.
(i) Name the phenomenon taking place when this happens.
(ii) Why does the sound become very loud for this length of the water column?
(c)
(i) What is meant by the terms (1) amplitude (2) frequency, of a wa e?
(ii) Explain why stringed musical instruments, like the guitar, are provided with a
hollow box.
Question 8 [10]
(a)
(i) It is observed that the temperature of the surroundings starts falling when the
ice in a frozen lake starts melting. Give a reason for the observation.
(ii) How is the heat capacity of the body related to its specific heat capacity?
(b)
(i) Why does a bottle of soft drink cool faster when surrounded by ice cubes than by
ice cold water, both at 0o C?
(ii) A certain amount of heat Q will warm 1 g of material X by 3o C and 1 g of material
Y by 4o C. Which material has a higher specific heat capacity?
(c) A calorimeter of mass 50 g and specific heat capacity 0.42 J g-1 oC-1 contains some
mass of water at 20o C. A metal piece of mass 20 g at 100 oC is dropped into the
calorimeter. After stirring, the final temperature of the mixture is found to be 22o C.
Find the mass of water used in the calorimeter.
[Specific heat capacity of the metal piece = 0.3 J g-1 oC-1
Specific heat capacity of water = 4.2 J g-1 oC-1]

Question 9 [10]
(a)
(i) State Ohm’s law.
(ii) A metal wire of resistance 6  is stretched so that its length is increased to twice
its original length. Calculate its new resistance.
(b)
(i) An electrical gadget can give an electric shock to its user under certain
circumstances. Mention any two of these circumstances.
(ii) What preventive measure provided in a gadget can protect a person from an
electric shock?
(c) The figure shows a circuit

When the circuit is switched on, the ammeter reads 0.5 A.


(i) Calculate the value of the unknown resistor R.
(ii) Calculate the charge passing through the 3  resistor in 120 s.
(iii) Calculate the power dissipated in the 3  resistor.
Question 10 [10]
(a) Name the three main parts of Cathode Ray Tube.
(b)
(i) What is meant by Radioactivity?
(ii) What is meant by nuclear waste?
(iii) Suggest one effective way from the safe disposal of nuclear waste.
(c)
(i) Draw a simple labelled diagram of a d.c. electric motor.
(ii) What is the function of the split rings in a d.c. motor?
(iii)State one advantage of a.c. over d.c.
ICSE Class 10 Physics Previous Year Question Paper
2010

PHYSICS
Science Paper - 1

Maximum Marks: 80 Time: One hour and a half

1. Answer to this Paper must be written on the paper provided separately.


2. You will not be allowed to write during the first 15 minutes. This time is to be
spent in reading the Question Paper.
3. The time given at the head of this Paper is the time allowed for writing the
answers.
4. Section I is compulsory. Attempt any four questions from Section II.
5. The intended marks for questions or parts of questions are given in brackets [ ].

SECTION-I (40 Marks)

(Attempt all questions from this Section)

Question 1.

(a) Name the device used for measuring:


(i) mass
(ii) weight. [2]

(b) A boy weighs 360 N on the earth : [2]


(i) What would be his approximate weight on the moon?
(ii) What is the reason for your answer?

(c) A body is acted upon by a force. State two conditions under which the
work done could be zero. [2]

(d) A spring is kept compressed by a small trolley of mass 0.5 kg lying on


a smooth horizontal surface as shown in the figure given below :

When the trolley is released, it is found to move at a speed of 2 ms-1.


What potential energy did the spring possess when compressed? [2]

(e) Name the subjective property :


(i) of sound related to its frequency.
(ii) of light related to its wavelength. [2]

Question 2.
(a) (i) Why is the mechanical advantage of a lever of the second order
always greater than one?
(ii) Name the type of single pulley that has a mechanical advantage
greater than one? [2]

(b) (i) What is meant by refraction of light?


(ii) What is the cause of refraction of light? [2]

(c) ‘The refractive index of diamond is 2.42’.


What is meant by this statement? [2]

(d) We can burn a piece of paper by focusing the sun rays by using a
particular type of lens.
(i) Name the type of lens used for the above purpose.
(ii) Draw a ray diagram to support your answer. [2]

(e) A ray of light enters a glass slab PQRS, as shown in the diagram. The
critical angle of the glass is 42o. Copy this diagram and complete the
path of the ray till it emerges from the glass slab.

Mark the angles in the diagram wherever necessary. [2]

Question 3.
(a) State two differences between light waves and sound waves. [2]

(b) Two waves of the same pitch have their amplitudes in the ratio 2:3.
(i) What will be the ratio of their loudness?
(ii) What will be the ratio of their frequencies? [2]

(c) Give two differences between a d.c. motor and an a.c. generator. [2]

(d) Six resistances are connected together as shown in the figure. Calculate
the equivalent resistance between the points A and B. [2]

(e) (i) Which part of an electrical appliance is earthed ?


(ii) State a relation between electrical power, resistance and potential
difference in an electrical circuit. [2]

Question 4.

(a) A device is used to transform 12V a.c. to 200 V a.c.


(i) What is the name of this device ?
(ii) Name the principle on which it works. [2]

(b) (i) Which material is the calorimeter commonly made of ?


(ii) Give one reason for using this material. [2]

(c) (i) Name a metal that is used as an electron emitter.


(ii) Give one reason for using this metal. [2]
(d) Complete the following nuclear changes:
11 Na ..... Mg  10 
24 .....
(i)
(ii) 238
92 U  234
90 Th  .............. Energy [2]

(e) (i) Which radiation produces maximum biological damage ?


(ii) What happens to the atomic number of an element when the
radiation named by you in part (i) above, are emitted? [2]

Section-II (40 Marks)


(Attempt any four questions from this Section)
Question 5.

(a) (i) Define the term momentum.


(ii) How is force related to the momentum of a body?
(iii) State the condition when the change in momentum of a body
depends only on the change in its velocity. [3]

(b) A body of mass 50 kg has a momentum of 3000 kg ms-1. Calculate:


(i) the kinetic energy of the body.
(ii) the velocity of the body. [3]

(c) (i) Write a relation expressing mechanical advantage of a lever?


(ii) Write an expression for the mechanical advantage of an inclined
plane.
(iii) Give two reasons as to why the efficiency of a single movable
pulley system is always less than 100%. [4]

Question 6.

(a) A stick partly immersed in water appears to be bent. Draw a ray


diagram to show the bending of the stick when placed in water and
viewed obliquely from above. [3]

(b) A ray of monochromatic light is incident from air on a glass slab:


(i) Draw a labelled ray diagram showing the change in the path of
the ray till it emerges from the glass slab.
(ii) Name the two rays that are parallel to each other.
(iii) Mark the lateral displacement in tour diagram. [3]

(c) An erect, magnified and virtual image is formed, when an object is


placed between the optical centre and principal focus of a lens.
(i) Name the lens.
(ii) Draw a ray diagram to show the formation of the image with
the above stated characteristics. [4]
Question 7.

(a) Two parallel rays of Red and Violet travelling through air, meet the air-
glass boundary as shown in the above figure:
(i) Will their paths inside the glass be parallel?
Give a reason for your answer.
(ii) Compare the speeds of the two rays inside the glass. [3]

(b) (i) A man stands at a distance of 68 m from a cliff and fires a gun.
After what time interval will he hear the echo, if the speed of
sound in air is 340 ms-1?
(ii) If the man had been standing at a distance of 12 m from the cliff
would he have heard a clear echo? [3]

(c) (i) In what unit does the domestic electric meter measure the electrical
energy consumed ? State the value of this unit in S.I. Unit.
(ii) Why should switches always be connected to the live wire?
(iii) Give one precaution that should be taken while handling switches.
[4]

Question 8.

(a) Calculate the quantity of heat that will be produced in a coil of


resistance 75Ω if a current of 2A is passed through it for 2 minutes.
[3]

(b) (i) A substance has nearly zero resistance at a temperature of 1K.


What is such a substance called?
(ii) State any two factors which affect the resistance of a metallic
wire. [3]
(c) Five resistors of different resistances are connected together as shown in the
figure. A 12 V battery is connected to the arrangement. Calculate:

(i) the total resistance in the circuit.


(ii) the total current flowing in the circuit. [4]

Question 9.

(a) (i) Define the term ‘specific latent heat of fusion’ of a substance.
(ii) Name the liquid which has the highest specific heat capacity.
(iii) Name two factors on which the heat absorbed or given out by a body
depends. [3]

(b) (i) An equal quantity of heat is supplied to two substances A and B. The
substance A shows a greater rise in temperature. What can you say
about the heat capacity of A as compared to that of B?

(ii) What energy change would you expect to take place in the molecules
of a substance when it undergoes:
1. a change in its temperature ?
2. a change in its state without any change in its temperature? [3]

(c) 50 g of ice at 0oC is added to 300 g of a liquid at 30oC. What will be the final
temperature of the mixture when all the ice has melted ? The specific heat
capacity of the liquid is 2.65 J g-1 oC-1 while that of water is 4.2 J g-1 oC-1.
Specific latent heat of fusion of ice =336 J g-1. [4]

Question 10.
(a) (i) Name the radioactive radiations which which have the least
penetrating power.
(ii) Give one use of radio isotopes.
(iii) What is meant by background radiation ? [3]
(b) (i) A straight wire conductor passes vertically through a piece of
cardboard sprinkled with iron filings. Copy the diagram and show the
Setting of iron filings when a current is passed through the wire in the
upward direction and the cardboard is tapped gently. Draw arrows to
represent the direction of the magnetic field lines. [3]

(ii) Name the law which helped you to find the direction of the magnetic field
lines.

(c) (i) State two ways by which the magnetic field of a solenoid can be made
stronger.
(ii) What material is used for making the armature of an electric bell? Give a
reason for using this material. [4]
ICSE Class 10 Physics Previous Year Question Paper
2012
PHYSICS
Science Paper - 1
Time: 1½hrs. Maximum Marks: 80

General Instruction
1. Answer to this Paper must be written on the paper provided separately.
2. You will not be allowed to write during the first 15 minutes. This time is to be spent
in
3. reading the Question Paper.
4. The time given at the head of this Paper is the time allowed for writing the answers.
5. Section I is compulsory. Attempt any four questions from Section II.
6. The intended marks for questions or parts of questions are given in brackets [ ].

Section - I
Question 1 [10]
Question 2 [10]
Question 3 [10]

Question 4 [10]
Section- II
Question 5 [10]
Question 6 [10]
Question 7 [10]
Question 8 [10]

Question 9 [10]
Question 10 [10]
ICSE Class 10 Physics Previous Year Question
Paper 2014
Physics
SCIENCE Paper - 1
(Two hours)
Answers to this Paper must be written on the paper provided separately.
You will not be allowed to write during the first 15 minutes.
This time is to be spent in reading the Question Paper.
The time given at the head of this Paper is the time allowed for writing the answers.

Section I is compulsory. Attempt any four questions from Section II.


The intended marks for questions or parts of questions are given in brackets [ ].

SECTION I (40 Marks)


Attempt all questions from this Section.

Question 1

(a) A force is applied on (i) a non-rigid body and (ii) a rigid body. How does the effect
of the force differ in the above two cases? [2]
(b) A metallic ball is hanging by a string from a fixed support. Draw a neat labelled
diagram showing the forces acting on the ball and the string. [2]
(c) (i) What is the weight of a body placed at the centre of the earth?
(ii) What is the principle of an ideal machine? [2]
(d) Is it possible to have an accelerated motion with a constant speed? Explain [2]
(e) (i) When does a force do work?
(ii) What is the work done by the moon when it revolves around the earth? [2]

Question 2

(a) Calculate the change in the Kinetic energy of a moving body if its velocity is reduced
to l/3rd of the initial velocity. [2]
(b) State the energy changes in the following devices while in use:
(i) A loud speaker.
(ii) A glowing electric bulb. [2]
(c) (i) What is nuclear energy?
(ii) Name the process used for producing electricity using nuclear energy. [2]
(d) State one important advantage and disadvantage each of using nuclear energy for
producing electricity. [2]
(e) (i) The conversion of part of the energy into an undesirable form is called
.
(ii) For a given height h, the length l of the inclined plane, lesser
will be the effort required. [2]

Question 3

(a) Draw the diagram given below and clearly show the path taken by the emergent
ray.

[2]

(b) (i) What is consumed using different electrical appliances, for which electricity
bills are paid?
(ii) Name a common device that uses electromagnets. [2]
(c) (i) A ray of light passes from water to air. How does the speed of light change?
(ii) Which colour of light travels fastest in any medium except air? [2]
(d) Name the factors affecting the critical angle for the pair of media. [2]
(e) (i) Name a prism required for obtaining a spectrum of Ultraviolet light.
(ii) Name the radiations which can be detected by a thermopile. [2]
Question 4

(a) Why is the colour red used as a sign of danger? [2]


(b) (i) What are mechanical waves?
(ii) Name one property of waves that do not change when the wave passes from
one medium to another. [2]
(c) Find the equivalent resistance between points A and B

[2]

(d) 50 g of metal piece at 27C requires 2400 J of heat energy so as to attain a


temperature of 327 °C . Calculate the specific heat capacity of the metal. [2]
(e) An electron emitter must have work function and melting
point. [2]

SECTION II (40 Marks)


Attempt any four questions from this Section

Question 5

(a) (i) A man having a box on his head, climbs up a slope and another man
having an identical box walks the same distance on a levelled road.
Who does more work against the force of gravity and why?
(ii) Two forces each of 5N act vertically upwards and downwards respectively on
the two ends of a uniform metre rule which is placed at its mid-point as shown in
the diagram. Determine the magnitude of the resultant moment of these forces
about the midpoint. [4]
(b) (i) A body is thrown vertically upwards. Its velocity keeps on decreasing.
What happens to its kinetic energy as its velocity becomes zero?
(ii) Draw a diagram to show how a single pulley can be used so as to have
its ideal M.A= 2. [3]
(c) Derive a relationship between mechanical advantage, velocity ratio and
efficiency of a machine. [3]

Question 6

(a) (i) Light passes through a rectangular glass slab and through a triangular
glass prism. In what way does the direction of the two emergent beams differ and
why?
(ii) Ranbir claims to have obtained an image twice the size of the object with a
concave lens. Is he correct? Give a reason for your answer. [4]
(b) A lens forms an erect, magnified and virtual image of an object.
(i) Name the lens.
(ii) Draw a labelled ray diagram to show the image formation. Define the power of a
lens. [3]
(c) (i) Define the power of a lens.
(ii) The lens mentioned in 6(b) above is of focal length 25cm. Calculate
the power of the lens. [3]

Question 7

(a) The adjacent diagram shows three different modes of


vibrations P, Q and R of the same string.
(i) Which vibration will produce a louder sound and
why?
(ii) The sound of which string will have maximum
shrillness?
(iii) State the ratio of wavelengths of P and R. [4]
O
(b) A type of electromagnetic wave has wavelength 50 A .
(i) Name the wave.
(ii) What is the speed of the wave in vacuum?
(iii) State one use of this type of wave. [3]
(c) (i) State one important property of waves used for echo depth sounding.
(ii) A radar sends a signal to an aircraft at a distance of 30 km away and
receives it back after 2 x 10-4 second. What is the speed of the signal? [3]

Question 8

(a) Two resistors of 4Ω and 6 Ω are connected in parallel to a cell to draw 0.5 A current
from the cell.
(i) Draw a labelled circuit diagram showing the above arrangement.
(ii) Calculate the current in each resistor. What is an Ohmic resistor? [4]
(b) (i) What is an Ohmic resistor?
(ii) Two copper wires are of the same length, but one is thicker than the other.
(1) Which wire will have more resistance?
(2) Which wire will have more specific resistance? [3]
(c) (i) Two sets A and B, of three bulbs each, are glowing in two separate rooms.
When one of the bulbs in set A is fused, the other two bulbs also cease to glow.
But in set B, when one bulb fuses, the other two bulbs continue to glow. Explain
why this phenomenon occurs.
(ii) Why do we prefer arrangements of Set B for house circuiting? [3]

Question 9

(a) Heat energy is supplied at a constant rate to 1OOg of ice at 0 °C. The ice is converted
into water at 0° C in 2 minutes. How much time will be required to raise the
temperature of water from 0 °C to 20 °C? [Given: sp. heat capacity of water = 4.2 J g-
1 °C-1 , sp. latent heat of ice = 336 J g-1]. [4]
(b) Specific heat capacity of substance A is 3.8 J g-1K-1 whereas the specific heat capacity
of substance B is 0.4 J g-1 K-1
(i) Which of the two is a good conductor of heat?
(ii) How is one led to the above conclusion?
(iii) If substances A and B are liquids then which one would be more useful in
car radiators? [3]
(c) (i) State any two measures to minimize the impact of global warming.
(ii) What is the Greenhouse effect? [3]

Question 10

(a) (i) Name two factors on which the magnitude of an induced e.m.f. in the
secondary coil depends.
(ii) In the following diagram an arrow shows the motion of the coil towards the
bar magnet.
(1) State in which direction the current flows, A to B or B to A?
(2) Name the law used to come to the conclusion.

[4]
(b) A nucleus 11Na 24 emits a beta particle to change into Magnesium (Mg)
(i) Write the symbolic equation for the process.
(ii) What are numbers 24 and 11 called?
(iii) What is the general name of 24 Mg with respect to 24 Na ? [3]
12 11

(c) In a cathode ray tube state:


(i) the purpose of covering cathode by thorium and carbon.
(ii) the purpose of the fluorescent screen.
(iii) how is it possible to increase the rate of emission of electrons. [3]
ICSE Class 10 Physics Question
Paper Solution 2015
PHYSICS
SCIENCE PAPER I
Question 1
(a) When a body is placed on a table top, it exerts a force equal to its weight downwards on
the table top but does not move or fall.

Table top

Force due to weight of the body

(i) Name the force exerted by the table top.


(ii) What is the direction of the force? [2]
(b) (i) Name one factor that affects the lateral displacement of light as it passes through a
rectangular glass slab.
(ii) On reversing the direction of the current in a wire, the magnetic field produced by
it gets --------------. [2]
(c) (i) On what factor does the position of the centre of gravity of a body depend?

(ii) What is the S.I unit of the moment of force? [2]


(d) Name the factors affecting the turning effect of a body. [2]
(e) (i) Define equilibrium.
(ii) In a beam balance when the beam is balanced in a horizontal position, it is in
equilibrium. [2]
Examiners’ Comments
(a) (i) Most candidates answered correctly, however
some named the force incorrectly as force applied Suggestions for teachers
or force due to friction. - The topic on centre of gravity
(ii) Many candidates simply wrote the direction to be should be explained by giving a
upward rather than stating it to be vertically variety of examples so that students
upward. understand that even bodies of same
(b) Most candidates answered it correctly though some volumes or same mass can still
wrote changed instead of reversed for direction. have different positions of centre of
gravity.

138
(c) (i) The factors affecting the position of centre of
gravity of a body was mentioned incorrectly by - Highlight the difference between
many candidates. The factors stated were distance and perpendicular distance
incorrect as it dealt with size, volume or mass. while explaining factors affecting
(ii) The S.I unit of movement of force was incorrectly the turning effect.
answered by many candidates. - Train students to read the question
(d) Answered correctly by most candidates though many carefully to enable correct answers
overlooked the importance of mentioning the being written.
perpendicular distance of the line of action of force - Important keywords of the
from the axis of rotation. definition should be stressed on.
(e) Most candidates stated the condition for equilibrium - Insist on students reading the
rather than defining equilibrium. question carefully and
understanding exactly what is
expected as an answer.
- Highlight the difference between
upwards and vertically upwards
while teaching the direction of
reaction forces.
- Explain that the word changed will
imply the change in
magnitude/direction/both whereas
reversed is used only for direction.

MARKING SCHEME
Question 1
(a) (i) Normal reaction force
(ii) Vertically upwards.
(b) (i) Thickness of glass slab / angle of incidence/refractive index of glass/colour of light/
wave length of light ( Any one)
(ii) reversed in direction
(c) (i) Shape of body/distribution of mass of body.
(ii) Nm.
(d) (i) magnitude of the force applied .
(ii) Perpendicular distance of line of action of the force from the axis of rotation./ Moment
arm / perpendicular distance between point of application of force and point of rotation
(Any one)
(e) (i) When a number of forces acting on a body produce no change in its state of rest or
motion, the body is said to be in equilibrium.
(ii) Static.

139
Question 2
(a) How is work done by a force measured when the force :
(i) is in the direction of displacement
(ii) is at an angle to the direction of displacement. [2]
(b) State the energy changes in the following while in use:
(i) Burning of a candle.
(ii) A steam engine. [2]
(c) (i) A scissor is a multiplier.
(ii) 1 kWh = J. [2]
(d) Explain the motion of a planet around the sun in a circular path. [2]
(e) Rajan exerts a force of 150 N in pulling a cart at a constant speed of 10 m/s. [2]
Calculate the power exerted.
Examiners’ Comments
(a) Answers were generally correct if stated in terms of
equations but misleading and contradictory when Suggestions for teachers
 If θ=0^othen Cos 0o =1 and hence
described in words. Some candidates used vague
symbols, which were irrelevant. W=FS. Also when θ is at an angle
(b) Most candidates answered this question correctly. less than 90o then W=FS Cosθ
Some however made errors in the sequence of needs to be made clear to the
energy conversions. students so that the concept is
(c) Relationship between kWh and Joules was stated understood correctly.
 Discuss examples of energy
incorrectly as the relationship between kWh and
conversions.
mega joule.
 Students should be trained to state
(d) Inappropriate explanations were written by many
important energy changes and in a
candidates. They should have explained that for
proper sequence.
planetary motion the gravitational pull of the sun
 Students should avoid selective
provides the necessary centripetal and not
study and attach importance to all
centrifugal force. Many candidates wrote answers
topics.
which conveyed the meaning partially.
 Train students to use the correct
(e) Many candidates did not seem to be familiar with
the formula P=F.v. Hence, a variety of methods language which can explain the
were tried. Incorrect unit of power was stated by phenomenon in a comprehensive
many candidates. way.
 Additional practice on numerical
problems needs to be given.
Students need to be trained to
present the answer with the correct
unit.

140
MARKING SCHEME
Question 2.
(a) (i) W = F X S
(ii) W = F X S Cos θ.
(b) (i) Chemical energy to Light energy and heat energy.
(ii) Chemical energy to Heat energy to Mechanical energy
(c) (i) Speed
(ii) 3.6 x 106 J.
(d) A planet moves around the sun in a nearly circular path for which the gravitational
force of attraction on the planet by the sun provides the centripetal force.
(e) P=FXV
= 150 X 10
= 1500 W

Question 3
(a) (i) Give the expression for mechanical advantage of an inclined plane in terms of the
length of an inclined plane.
(ii) Name a common device where a gear train is used. [2]
5
(b) The speed of light in glass is 2 x 10 km/s. What is the refractive index of glass? [2]
(c) (i) Draw a graph between displacement and the time for a body executing free
vibrations.
(ii) Where can a body execute free vibrations? [2]
(d) (i) What happens to the resistivity of semi-conductors with the increase of
temperature?
(ii) For a fuse, higher the current rating is the fuse wire. [2]
(e) (i) Name the high energetic invisible electromagnetic waves which help in the study
of the structure of crystals.
(ii) State an additional use of the waves mentioned in part (e) (i). [2]

141
Examiners’ Comment
(a)(i) Candidates were unable to understand the
question and hence many of them wrote Suggestions for teachers
- Students must answer the question
MA= or . Some even wrote that MA=
asked rather than write an
(ii) Many candidates did not seem to be familiar with unconnected detailed version.
a common device using a gear train. - It is advisable to write simple
(b) Some candidates while solving did not convert the examples rather than irrelevant ones.
speed in the same units and hence obtained incorrect - Stress on using data in the same
answers. units while solving numerical
(c) (i) The negative displacement was not shown by problems.
many candidates. Some candidates drew graphs - Reading and understanding of
which had a decreasing amplitude. graphs should be given greater
(ii) Most candidates did not understand that the weightage.
“where” in the question meant the medium and - While teaching conductors the
hence answered incorrectly. topics of semi-conductors and super
(d) Effect of temperature on resistivity of semi- conductors could be taken up
conductors was not known by many candidates. simultaneously so as to help in a
Some treated semi-conductors as conductors and better understanding.
hence wrote incorrect answers. - Teach students to identify the
(e) Most candidates were unable to identify the X-rays electromagnetic radiations by
correctly. wavelength; uses and characteristics.
- Wavelength ranges for different
radiations should be taught and
candidates made to revise regularly.
- While teaching ensure that uses and
properties of waves are related to
each other rather than teaching them
independently.

MARKING SCHEME
Question 3.
(a) (i) M.A. = l/h
(ii) It is used in a clock or a watch
(b) Refractive index of glass = speed of light in vacuum / speed of light in glass
= 3 X 108 / 2 X 105 X 1000
= 1.5

142
(c) (i)
a

-a time

(ii) in Vacuum.
(d) (i) Resistivity decreases with the increase of temperature
(ii) Thicker
(e) (i) X rays
(ii) to detect the fracture in bones.

Question 4
(a) Rishi is surprised when he sees water boiling at 115⁰ C in a container. Give reasons as to [2]
why water can boil at the above temperature.

(b) (i) Why does a current carrying, freely suspended solenoid rest along a particular
direction?
(ii) State the direction in which it rests. [2]
(c) Find the equivalent resistance between points A and B.

2Ω
A

6Ω 4Ω
12Ω

B [2]
5Ω
(d) Give two similarities between an A.C generator and a D.C motor. [2]
(e) (i) Why is a cathode ray tube evacuated to a low pressure?
(ii) What happens if the negative potential is changed on a grid? [2]

143
Examiners’ Comments
(a) Most candidates wrote the correct reason but were Suggestions for teachers
unable to explain it further on the basis of the  Advise students to revise the topic on
stated reason. ‘HEAT’ thoroughly.
(b) Most candidates were able to understand that the  Guide students to explain the
solenoid gets magnetized but missed out that the dependence on the stated factor and
solenoid starts behaving like a bar magnet and not on some other factor.
hence were unable to state the direction in which it  Solenoid is an important topic and
rests. hence it should be thoroughly
(c) For parallel combination of equivalent resistance explained that when current passes
many candidates wrote Rp instead of in the through it, it behaves like a bar
L.H.S of the equation. No unit was written in the magnet.
final answer.  Students need to be given adequate
(d) Basic similarities between the two were not clearly practice in solving numerical
understood by many candidates. Some wrote problems that should be
differences instead of similarities. mathematically correct.
(e) Many candidates were unable to comprehend the  The AC generator and DC motor
question as they were not aware of the specific ought to be taught simultaneously so
functions of various parts in a C.R.T. that students are aware of the basic
differences and similarities in their
working.
 Through the help of a labelled
diagram of a C.R.T, explain the
various parts and their specific
functions rather than teaching it as a
whole.

MARKING SCHEME
Question 4.
(a) Because of the presence of impurity. ( salt )
More the impurity more will be the boiling point.
(b) (i) A current carrying freely suspended solenoid behaves like a bar magnet
(ii) Geographic north south direction
(c) 1/R = 1/R1 + 1/ R2 + 1/ R3
= 1/12 + 1/6 + 1/4
= 6 / 12
= 1/ 2
R=2Ω
R = 2 + 2 + 5 = 9 Ω

144
(d) Both in an A.C generator and D.C motor a coil rotates in magnetic field between
the pole pieces of a powerful magnet / presence of carbon brushes/ presence of armature coil
(any two)
(e) (i) to avoid collisions of electrons with the air molecules
(ii) The brightness of the pattern on the screen can be changed.

Question 5
(a) Draw a simplified diagram of a lemon crusher, indicating direction of load and effort. [2]
(b) (i) Name the physical quantity measured in terms of horse power.
(ii) A nut is opened by a wrench of length 20cm. If the least force required is 2N, find
the moment of force needed to loosen the nut.
(iii) Explain briefly why the work done by a fielder when he takes a catch in a cricket [4]
match is negative.
(c) A block and tackle system has V.R. = 5.
(i) Draw a neat labelled diagram of a system indicating the direction of its load and
effort.
(ii) Rohan exerts a pull of 150 Kgf. What is the maximum load he can raise with this [4]
pulley system if its efficiency = 75%?
Examiners’ Comments
(a) Candidates paid more attention to drawing an actual
Suggestions for teachers
lemon crusher instead of drawing a simplified
 Students should be trained to draw
diagram. Direction of the load and effort either was
the schematic diagram of levers and
not shown or drawn correctly.
not waste time over the actual
(b) Calculation errors were common as candidates did not
pictorial diagrams. Emphasis must
convert the length to S.I unit. Many stated the answer
be put on the direction of load and
in mixed units. Answer in part (iii) was neither
effort and the relative position of
explained in terms of force and displacement nor any
fulcrum load and effort.
mention of 180o as the angle between the both was
 Students need to be advised to
stated.
write the answer with the correct
(c) Many candidates were unable to draw the diagram
unit and given sufficient practice in
correctly showing the support and the direction of
solving numerical problems.
load and effort. In part (ii) errors were made in
 Concept of negative work needs to
calculation and mathematical presentation. A few
be explained clearly with adequate
candidates took MA as 5 and missed out that
examples.
efficiency was given as 75%.
 Emphasize on the drawing of
support and marking of the
direction of the various forces in
the pulley system.
 Inculcate the habit of writing the
formula or equation while solving
numerical problems.

145
MARKING SCHEME
Question 5.
(a) Lemon crusher :
(b) (i) Power of the appliance.
(ii) Moment of force = Force x Moment arm
= 2 x 20/100
= 0.4Nm
(iii) Force exerted on the ball and the displacement of the ball are in opposite
directions or they make 1800 with one another.
(c) (i) Correct diagram drawn with directions of load and effort shown correctly
(ii) Efficiency = M.A / V.R
75 / 100 = M.A / 5 M.A = 3 .75
M.A = Load / Effort
3 .75 = Load / 1500 or 3.75 = Load / 150
Load = 5625 N or Load = 562.50 kgf

Question 6
(a) (i) Where an object should be placed so that a real and inverted image of the same size
as the object is obtained using a convex lens?
(ii) Draw a ray diagram to show the formation of the image as specified in the part a (i) [3]
(b) (i) Why does the Sun appear red at sunrise?
(ii) Name the subjective property of light related to its wavelength. [3]
(c) Jatin puts a pencil into a glass container having water and is surprised to see the pencil in
a different state.
(i) What change is observed in the appearance of the pencil?
(ii) Name the phenomenon responsible for the change.
(iii) Draw a ray diagram showing how the eye sees the pencil. [4]

146
Examiners’ Comments
(a) Many candidates wrote F2 in place of 2F. The
Suggestions for teachers
following errors were observed in the ray diagram-
 Adequate practice of drawing
1. When the optical plane was represented by a
correct ray diagrams needs to be
straight line, it was not labelled.
given.
2. Incorrect arrows or arrows not marked.
 Distinguish between F1, F2, 2F1,
3. Real image shown by a dotted line.
2F2.
4. Actual path of the rays shown with dotted lines.
(b) Most candidates gave a confused explanation with  Insist that virtual rays and virtual
images should be represented
the word scattering being incorrectly addressed as
through dotted lines. All ray
dispersion, deviation, etc. The concept that the
diagrams need to be practiced
intensity of scattered radiation decreases with an
several times.
increase of wavelength was not clearly understood.
 A clear understanding of dispersion,
Many did not seem to understand the meaning of
‘subjective’.  deviation and scattering should be
(c) Many candidates wrote that the pencil will appear at  brought out by examples and natural
phenomenon.
a different place. Probably the phrase “different
 Students should be explained on the
state” in the question was misunderstood. In the
difference between subjective and
diagram the following errors were commonly
objective properties.
spotted:
1. Arrows not drawn or marked incorrectly.
2. Dotted lines were drawn at wrong places.
3. Incorrect bending of rays.
4. Incorrect image location.

MARKING SCHEME
Question 6.
(a) (i) At 2F1 or centre of curvature
(ii) A ray shown parallel to the principal axis and passing through F2.
A ray shown passing through optical centre and moving undeviated.
Both rays drawn correctly.
Real image shown correctly.
(b) (i) At sunrise light has to travel longest distance in atmosphere, blue light gets
Scattered more and is lost.
Red Light has long wavelength, scatters the least and reaches the observer.
(ii) Colour of light
(c) (i) The pencil will be seen bent.
(ii) Refraction of light.
(iii) Two rays correctly drawn.
(iv) Image shown correctly.

147
Question 7
(a) (i) State the safe limit of sound level in terms of decibel for human hearing.
(ii) Name the characteristic of sound in relation to its waveform. [2]
(b) A person standing between two vertical cliffs and 480 m from the nearest cliff
shouts. He hears the first echo after 3s and the second echo 2s later.
Calculate:
(i) The speed of sound.
(ii) The distance of the other cliff from the person. [3]
(c) In the diagram below, A, B, C, D are four pendulums suspended from the
same elastic string PQ. The length of A and C are equal to each other while
the length of pendulum B is smaller than that of D. Pendulum A is set into a
mode of vibrations.

P Q

A C
D
(i) Name the type of vibrations taking place in pendulums B and D?
(ii) What is the state of pendulum C?
(iii) State the reason for the type of vibrations in pendulums B and C. [5]
Examiners’ Comments
(a) Many candidates misunderstood the question and Suggestions for teachers
wrote the audible range of human hearing in terms of - Questions must be read carefully
frequency rather than the safe limit of sound in terms before attempting to answer them.
of decibel. Characteristics of sound not very clear to
- Safe limit of sound level should be
many candidates. explained clearly on the basis of
(b) Many candidates displayed lack of practice in solving
loudness in terms of decibel.
numerical problems. Units either not mentioned or - Clear explanation of loudness,
mentioned incorrectly.
intensity, pitch and quality needs
(c) In part (i) many candidates wrote resonance and part to be given.
(ii) as forced vibrations. Many did not seem to be
- Additional practice in solving
aware that same length means same natural
numerical problems must be
frequencies. allotted. Encourage students to
write the formula used and stress
on correct S.I units.
- Thorough revision of resonance
and forced vibration with
examples must be conducted.

148
MARKING SCHEME
Question 7.
(a) (i) Safe limit of sound level - upto 120 db
(ii) Quality of sound waves
(b) (i) V = 2 distance / time
= 2 X 480 / 3
= 320 m / s
(ii) Distance of the other cliff = (Velocity X Time ) / 2
= ( 320 X5) / 2 = 800 m

(c) (i) Pendulum B and D execute forced vibrations


(ii) Pendulum C will be in a state of Resonance
(iii) Natural frequency of B does not match with Natural frequency of A
Natural frequency of C matches that of A
Question 8

(a) (i) Name the device used to increase the voltage at a generating station.
(ii) At what frequency is A.C supplied to residential houses?
(iii) Name the wire in a household electrical circuit to which the switch is connected. [3]
(b) The relationship between the potential difference and the current in a conductor is stated
in the form of a law.
(i) Name the law.
(ii) What does the slope of V- I graph for a conductor represent?
(iii) Name the material used for making the connecting wire. [3]
(c) A cell of Emf 2V and internal resistance 1.2 Ω is connected with an ammeter of
resistance 0.8 Ω and two resistors of 4.5 Ω and 9 Ω as shown in the diagram below:

2 

1.2 

4.5 
Ammeter

0.8 
9 
(i) What would be the reading on the Ammeter?

(ii) What is the potential difference across the terminals of the cell? [4]

149
Examiners’ Comments
(a) An incorrect name of the device was stated by a few
Suggestions for teachers
candidates. Many wrote the frequency of A.C supplied
 Both voltage and frequency must
as 220V in place of 50Hz. Some candidates wrote that
be clearly explained while
the switch is connected to live and neutral both.
teaching A.C.
(b) The law was correctly identified by most candidates  Clearly explain the characteristics
though for part (ii) many gave the answer as 1/R or V required by a connecting wire,
I or is a straight line. Many candidates wrote nichrome, resistance wire, etc.
manganin or tungsten as the material used that was  The concept of ‘slope’ must be
incorrect. made clear in case of a graph.
(c) The concept of series and parallel resistance is not clear  Adequate practice of numerical
to many candidates. Some candidates considered 0.8Ω problems must be carried out
as a part of parallel resistance; hence the remaining
regularly in class.
working was incorrect too.

MARKING SCHEME
Question 8.
(a) (i) Step up transformer
(ii) 50 hertz
(iii) Live wire.
(b) (i) Ohm’s Law.
(ii) Resistance of the conductor.
(iii) Copper / Aluminium
(c) (i) 1/R=1/R1+1/R2
= 1 / 9 + 1 / 4.5
= 1/3 R=3Ω
i =E/R+r
= 2 / ( 3 + 1.2 + 0.8 )
= 0.4 A Ammeter reading 0.4 A
(ii) V = E – ir
= 2 – 0.4 X 1.2
= 1.52 V
Or V = iR
= o.4 x ( 3 + 0.8)
= 1.52 V

150
Question 9
(a) (i) Name a gas caused by the Greenhouse effect.
(ii) Which property of water makes it an effective coolant? [2]
(b) (i) Water in lakes and ponds do not freeze at once in cold countries. Give a reason in
support of your answer.
(ii) What is the principle of Calorimetry?
(iii) Name the law on which this principle is based.
(iv) State the effect of an increase of impurities on the melting point of ice. [4]

(c) A refrigerator converts 100 g of water at 20⁰ C to ice at -10⁰ C in 35 minutes.


Calculate the average rate of heat extraction in terms of watts.
Given: Specific heat capacity of ice = 2.1 J g-1 ⁰ C-1
Specific heat capacity of water = 4.2 J g -1 ⁰ C -1 [4]
Specific Latent heat of fusion of ice = 336 J g -1
Examiners’ Comments
(a) Most candidates wrote the correct answer, however
along with the correct option they also wrote an Suggestions for teachers
incorrect option. Many either wrote the incorrect - EVS topics must be given due
value of SHC of water or wrote incorrect units. importance in class discussions.
(b) Many candidates held high SHC of water to be - It is also necessary to talk about
responsible rather than high SLHC of fusion. Many awareness of environmental related
failed to specify “if the system is fully insulated” issues while teaching physics
part in the principle of calorimetry. Incorrect - When only one option is asked for in
answers were written by many in part (iv). the question, students must be
(c) Many candidates showed an incorrect working advised to refrain from writing a
with Heat equations formed incorrectly. large number of options as their
Conversion from minutes to seconds was also not answers.
shown in many cases. - Students should be advised to
memorize the values and units of
important physical constants.
- Sufficient practice of numerical
problems involving different types
should be given.
- Encourage a correct analysis of
numerical problems.
- Answers with correct S.I units
should be insisted on.

151
MARKING SCHEME
Question 9.
(a) (i) CO2 , Water vapours, methane and chloroflouro carbons.( Any one)
(ii) Water has high specific heat capacity/ Specific heat capacity of water is 4200J kg-1 K-1
or any correct value in correct unit
(b) (i) This is because the specific latent heat of fusion of ice is sufficiently high
(336 J/ g), so to freeze water, a large quantity of heat has to be withdrawn, hence it freezes
slowly.
(ii) If the system is fully insulated then
Heat energy lost by the Hot body = Heat energy gained by the Cold body.
(iii) Law of conservation of energy.
(iv) Increase of impurity decreases the m.pt. of ice.
(c) mcθ+mL+mcθ=PXt
100 X 4.2 X20 + 100 X 336 + 100 X 2.1 X 10 = P X 35 X 60
44100 = P X 2100
P = 21 W

Question 10
(a) (i) What is thermionic emission?
(ii) Name the unit in which the work function of a metal is expressed. [2]

+ -

(b) (i) Complete the diagram as given above by drawing the deflection of
radioactive radiations in an electric field.

(ii) State any two precautions to be taken while handling radioactive substances. [5]

152
(c) An atomic nucleus A is composed of 84 protons and 128 neutrons.
(i) The nucleus A emits an alpha particle and is transformed into nucleus B.
What is the composition of nucleus B?
(ii) The nucleus B emits a beta particle and is transformed into a nucleus C.
What is the composition of nucleus C?
(iii) Does the composition of nucleus C change if it emits gamma radiations? [3]
Examiners’ Comments
(a) Most candidates failed to understand that the
Suggestions for teachers
thermionic emission is due to the heat energy being
- Explain the difference between
imparted to the metallic surface. In place of an
thermionic and electronic emission.
electron volt candidates wrote joule.
- The unit of work function and its
(b) Many candidates drew incorrect diagrams with
relation with the S.I unit of the
either the radiations being labelled incorrectly or the
physical quantity which it measures
deflection shown incorrectly.
can be explained at this stage.
(c) Most candidates did not understand the changes
- Comparative property of radioactive
coming in with the emission of an alpha particle or a
radiations in an electric field should
beta particle. Many failed to write the composition
be discussed during classroom
of the nucleus or explain appropriately the number
teaching.
of protons and neutrons present in the nucleus after
- Stress must also be laid to teach the
the emission with a few unable to understand the
atomic structure in the class so as to
effect of Gamma emissions.
make it clearer.

MARKING SCHEME
Question 10.
(a) (i) Emission of free electrons from a metal surface when heat energy is imparted to it is called
the thermionic emission.
(ii) Electron Volt.

(b) (i) Deflection of Alpha and Beta shown correctly, Alpha deflection is less than beta Path of
Gamma.
(ii) Use of lead lined aprons/ Lead gloves / long handle lead tongs/ radioactive substance
should be kept in a thick walled lead container (Any two)

208
(c) (i) 82B or 82 protons and 126 neutrons
208
(ii) 83C or 83 protons and 125 neutrons
(iii) No.

153
Topics/Concepts found difficult or confusing
 Cases of inter conversion of energy.
 Concept of work. Work done for different angles between force and displacement.
 Numerical problems based on p=f.v and machines.
 Interpretation of graphs and slopes.
 Vibration in a stretched string.
 Cases of resonance, forced vibrations, free vibrations, etc.
 Safe limit of sound level for humans.
 Numerical problems on “Heat”
 Difference between heat capacity, SHC and SLHC.
 Similarities between A.C generator and D.C motor.
 Cathode ray tube.
 Mass number and atomic number.

Suggestions for students


 Use the reading time of 15 minutes judiciously to make a proper choice of questions from section
II by reading the requirements of the question carefully supported with a high degree of
concentration.
 Avoid writing answers which are simply a repetition of the question. Instead be specific about the
key word in that statement.
 Candidates must not leave any topic for option. All topics are covered in section I which is
compulsory.
 Avoid changing the order of sequence of questions and numbering system.
 Handwriting should be neat and legible.
 Learn the principles, laws and definitions accurately.
 Ray diagrams and the other diagrams need to be practiced periodically.
 While writing the answers it is not only important to cover all the points but also to present them in
a proper sequence.
 While solving a numerical it is advisable that the formula needs to be written in the beginning.
Essential steps need to be shown and final answer needs to be expressed along with a proper unit.
Avoid computation at the first step; let it be plain substitution as the marks are awarded for the
correct substitution.
 A thorough revision of all topics is all time important.
 It is advisable to solve previous year’s papers in writing.
 More emphasis should be given on writing rather than memorizing.
 For speed in mathematical calculations; it is advisable to learn tables up to 30, know squares up to
30, cubes up to 15 and basics of fractions and decimal.
 It is better to inculcate the habit of underlining the important points or key words in the answer.

154
ICSE Class 10 Physics Question
Paper Solution 2016
PHYSICS
SCIENCE Paper – 1
I. ANALYSIS OF PERFORMANCE
Question 1
(a) (i) Give an example of a non-contact force which is always of attractive nature. [2]
(ii) How does the magnitude of this non-contact force on the two bodies depend on the
distance of separation between them?
(b) A boy weighing 40kgf climbs up a stair of 30 steps each 20cm high in 4 minute and a [2]
girl weighing 30kgf does the same in 3 minutes. Compare:-
(i) The work done by them.
(ii) The power developed by them.
(c) With reference to the terms Mechanical Advantage, Velocity Ratio and efficiency of a [2]
machine, name and define the term that will not change for a machine of a given design.
(d) Calculate the mass of ice required to lower the temperature of 300g of water at 40°C to [2]
water at 0°C.
(Specific latent heat of ice=336 J/g, Specific heat capacity of water=4.2 J/goC)
(e) What do you understand by the following statements: [2]
(i) The heat capacity of the body is 60 JK-1.
(ii) The specific heat capacity of lead is 130 Jkg-1K-1.

Comments of Examiners
Suggestions for teachers
a) Most candidates identified it correctly as  Students must be taught in what way
gravitational force however some wrote magnetic is the gravitational force different
or electrostatic force. Many identified the inverse from other non-contact forces. There
relation between force and distance but failed to is a need to emphasize on inverse
identify the square relation. square relation rather than saying
b) Most candidates failed to convert kgf to newton ‘one increases and the other
and from cm to m that resulted in wrong answers. decreases’.
Some showed the working correctly but failed to  Conversion from kgf to N should be
do the comparison and missed on scoring. emphasized periodically. Students
c) Majority of candidates failed to comprehend this should be made aware that when the
question. They wrote the definitions of M.A., question says compare then the
V.R. and efficiency without mentioning V.R. does answer should be in the form of ratio
or greater than or less than. (not in the
form of a fraction).

136
not change. Some They also wrote the definition of
V.R. in terms of displacements without stating that  Explain how V.R. is the same in
these displacements are in the same time. spite of varying M.A and efficiency
d) Answered correctly by most candidates. Some used for the same machine through
the value of S.H.C. of water in a wrong unit. final numerous examples.
answers in some cases had no mention of unit.  Students should be trained to read
e) Definitions lacked a clear understanding by most the question carefully and to analyze
candidates as the same expression was used with accordingly.
unit given in the question. It makes it clear that the
definitions were done by rote learning.  Students should be trained to
substitute values in appropriate units
and to write units appropriately.
 Proper understanding of definitions
need to be emphasized.

MARKING SCHEME
Question 1.
(a) (i) Gravitational force
(ii) Magnitude of non-contact forces on the two bodies is inversely proportional to the
Square of the distance of separation between them.
(b) i) Work done by the boy = 40×10×6 = 2400J
Work done by the girl = 30×10×6 = 1800J
Therfore, Ratio = 2400:1800 = 4:3
2400
ii) Power of boy = = 10 watt
4×60
1800
Power of girl = = 10 watt
3×60
Therefore Ratio = 10:10=1:1
(c) (i) Velocity Ratio
(ii) Ratio of velocity of effort to the velocity of effort Or ratio of displacement of effort to the
displacement of load in the same time.
(d) 𝑚 × 336 = 300 × 4.2 × 40
300 × 4.2 × 40
m=
336
=150 g
(e) (i) 60 J of heat energy is required to raise the temperature of a body by 1K.
(ii) Lead requires 130 J of heat energy to raise the temperature of 1 Kg by 1K.

137
Question 2
(a) State two factors upon which the heat absorbed by a body depends. [2]
(b) A boy uses blue colour of light to find the refractive index of glass. He then repeats the [2]
experiment using red colour of light. Will the refractive index be the same or different in
the two cases? Give a reason to support your answer.
(c) Copy the diagram given below and complete the path of light ray till it emerges out of [2]
the prism. The critical angle of glass is 42°. In your diagram mark the angles wherever
necessary. A
60o
P

Q
60o 60o
B C
(d) State the dependence of angle of deviation: [2]
(i) On the refractive index of the material of the prism.
(ii) On the wavelength of light.
(e) The ratio of amplitude of two waves is 3 : 4. What is the ratio of their: [2]
(i) loudness?
(ii) frequencies?

138
Comments of Examiners
a) Answered correctly by most candidates. However Suggestions for teachers
some stated only temperature instead of change in  Ensure the difference between 𝑇
temperature; some mentioned material and specific and ∆𝑇 as well as the difference
heat capacity as two different points; some even between heat absorbed and rate of
stated factors such as surface area and black surface absorption of heat is explained in
which are actually factors affecting the rate of heat detail.
absorption.  Explain the concept with difference
in deviation of different colours
b) Candidates failed to write that different colours have hence refractive index is different.
different refractive index. They were unable to make  Insist on drawing arrows on rays
the point clear that different colours have different before and after refraction or
speed in a different medium. reflection, on writing necessary
c) Most candidates drew this diagram correctly. Some angles and drawing normal on ray
however, failed to write the angle at the second and diagram by giving more practice of
at the third surface. ray diagrams.
d) Most candidates answered correctly.  Students should be made to
e) Some candidates did not square the amplitude understand the relation between
ratio; some incorrectly expressed ratio as a fraction. loudness and amplitude and also be
instructed not to express the ratio in
a fraction.

MARKING SCHEME
Question 2.
(a) -Directly proportional to the mass.
-Directly proportional to change in temperature.
-Directly proportional to the specific heat capacity. Any two factors.
(b) Refractive index will be different.
a𝜇g=
sin 𝑖⁄
sin 𝑟, different colours will have different angle of refraction even if the angle of
incidence is the same therefore refractive index will be different. OR Speed of red colour and
blue colour of light in air or vacuum is the same but in glass it is different. So refractive index of
𝑐
glass will be different as aµ g= 𝑎 .
𝑐𝑏

(c)

(d) (i)  of deviation decreases with the decrease in 

139
(ii)  of deviation decreases with the increase in 
(e) (i) 9 :16
(ii) 1:1

Question 3
(a) State two ways by which the frequency of transverse vibrations of a stretched string can [2]
be increased.
(b) What is meant by noise pollution? Name one source of sound causing noise pollution. [2]
(c) The V-I graph for a series combination and for a parallel combination of two resistors is [2]
shown in the figure below. Which of the two A or B, represents the parallel combination?
Give a reason for your answer.
Y
B
A
V
I X
(d) A music system draws a current of 400 mA when connected to a 12 V battery. [2]
(i) What is the resistance of the music system?
(ii) The music system is left playing for several hours and finally the battery voltage
drops and the music system stops playing when the current drops to 320 mA. At
what battery voltage does the music system stop playing?
(e) Calculate the quantity of heat produced in a 20 Ω resistor carrying 2.5A current in 5 [2]
minutes.
Comments of Examiners

140
a) Most candidates were unable to comprehend the
question and wrote incorrect answers. Some Suggestions for teachers
identified the factors but failed to give the correct  Explain the factors affecting
relation of these factors with the frequency. In many frequency in a stretched string using
cases the length was mentioned in direct proportion to sonometer.
the frequency. In some cases, students even related it  It is advisable to include following
to the overtones. points in definition of noise
b) Most candidates were unable to score as the loudness pollution: - sound above 120 dB and
limit 128 dB was not mentioned. Some wrote vague causing any physical discomfort such
examples of noise pollution as car, machine etc. as headache, ear pain etc. Students
need to be absolutely clear in giving
c) Most candidates identified the parallel combination examples.
but could not justify in terms of the slope of the  Students should keep in mind the
graph. concept from which they are coming
to the conclusion of the answer. In
d) Many candidates were unable to score as they did this question they should have given
not convert mA to A that led to loss of marks in the the answer in terms of slope rather
second part. Some candidates misinterpreted voltage saying that parallel combination
drop as potential drop and hence scored poorly. reduces resistance. The decision is
taken on the basis of slope.
e) Candidates knew the formula but during substitution  Necessary conversions and formulae
did not convert minutes to seconds. need to be drilled by giving lots of
practice of numerical. Students
should also be trained to give answer
with unit.

MARKING SCHEME
Question 3.
(a) i) By increasing the tension in the string
ii) By decreasing the length of the string.
iii) By decreasing the mass per unit length of the string Any two correct points
(b) The disturbance produced in the environment due to undesirable loud and harsh sound of level
above 120 dB.
Source: Honking of vehicles in traffic jams.
(c) // combination is represented by A.
As slope gives resistance and slope of A is less.
(d) R=V/I R = 12 / 400 x 10 -3
= 30 Ω
V=IR V = ( 320 x 10 -3) x 30

141
= 9.60 V
(e) H = 𝑖2Rt
= 2.5 × 2.5 × 20 × 300
= 37500 J
Question 4
(a) State the characteristics required of a good thermion emitter. [2]
(b) An element ZS decays to 85R after emitting 2 α particles and 1 β particle. Find the
A 222
[2]
atomic number and atomic mass of the element S.
(c) A radioactive substance is oxidized. Will there be any change in the nature of its [2]
radioactivity? Give a reason for your answer.
(d) State the characteristics required in a material to be used as an effective fuse wire. [2]
(e) Which coil of a step up transformer is made thicker and why? [2]

Comments of Examiners
Suggestions for teachers
a) Most candidates answered this correctly. However,  The difference between the melting
some answered good conductor of heat as one of the point and the boiling point must be
characteristics. Some related it to the factors affecting taught effectively.
the rate of emission from the metal surface. They were  Sufficient practice to find atomic no
confused with the d) part of the same question. and mass no of reactant by giving the
b) Most candidates answered correctly. A few however names of radiation and giving the
found it difficult to work backward to find the atomic atomic no. and mass no. of the
number and mass number. They either made calculation products must be given.
errors or interchanged the atomic no. and mass no. or  Students should be made clear about
forgot to consider the second alpha emission. the differences between nuclear and
c) Most candidates were able to answer the first part chemical changes.
correctly, but were unable to offer reasons.  Students should read the question
d) Answered correctly by most candidates. However,  carefully. Understand the working of
some wrote good conductor of heat instead of good  each component of the circuit.
resistor.  Demonstrate the working of primary
e) Some candidates identified the coil correctly but and secondary coil of transformer.
could not write the reason. Some wrote secondary coil Should clearly explain why thicker
or even wrote upper coil and lower coil. These errors wire is needed to carry a greater
showed lack of conceptual clarity on the working of a current.
transformer.

MARKING SCHEME
Question 4.
(a) Low work function
High M.P.

142
(b) A = 230
Z = 88
(c) No change
As radioactivity is a nuclear phenomenon.
(d) High Resistance
Low M.P.
(e) Primary Coil
Thicker wire reduces resistance which helps in reducing loss of energy.

SECTION II (40 Marks)


Attempt any four questions from this Section

Question 5
(a) A stone of mass ‘m’ is rotated in a circular path with a uniform speed by tying a strong [3]
string with the help of your hand. Answer the following questions:
(i) Is the stone moving with a uniform or variable speed?
(ii) Is the stone moving with a uniform acceleration? In which direction does the
acceleration act?
(iii) What kind of force acts on the hand and state its direction?
(b) From the diagram given below, answer the questions that follow: [3]

B
T
T T

A
C
L

(i) What kind of pulleys are A and B?


(ii) State the purpose of pulley B.
(iii) What effort has to be applied at C to just raise the load L=20kgf?
(Neglect the weight of pulley A and friction).

143
(c) (i) An effort is applied on the bigger wheel of a gear having 32 teeth. It is used to turn [4]
a wheel of 8 teeth. Where is it used?
(ii) A pulley system has three pulleys. A load of 120N is overcome by applying an effort
of 50N. Calculate the Mechanical Advantage and Efficiency of this system.

Comments of Examiners
Suggestions for teachers
a) Answered correctly by most candidates. The concept of uniform circular
b) Most candidates answered part i) correctly but some motion should be made clear to
could not state the purpose of pulley B. They stated the students alongwith centripetal and
use as to reduce the effort. Some could not do the centrifugal forces. The fact that the
calculation as they could not conclude the fact that ideal centrifugal force is not the reaction
M.A. of this pulley is 2 as weight and friction is absent. force to the centripetal force must be
A few lost marks due to careless errors (applied M.A.× emphasized on. Basically they should
L = E). be aware of the fact that the same
c) Some candidates wrote incomplete examples. They motion can be perceived differently if
have written in cars but failed to mention to multiply the observer is in a different frame of
speed in part i). Most answered correctly part ii) but reference.
made careless calculation error in calculating  Students need to understand the
efficiency. Some candidates have even done working advantages and disadvantages of fixed
by assuming one fixed and several movable pulleys. and movable pulley clearly. It may be
effectively demonstrated by an actual
arrangement of pulleys.
 Explain the functioning of gears with
demo lesson with the help of video
presentations. Explain students that
when the type of pulley system is not
mentioned then they should assume it
to be block and tackle system with
convenient direction.

MARKING SCHEME
Question 5.
(a) (i) Uniform speed
(ii) Yes, Radially inwards.
(iii) Centrifugal force acting radially outward
(b) (i) A is movable pulley and B is fixed pulley.
(ii) So as to apply the effort in a convenient direction that is vertically downwards.
20
(iii) M. A. = 2 = Therefore , E = 10kgf
𝐸
(c) (i) It is used to gain speed.
(ii) MA = L/E = 120/50 = 2.4

144
VR = 3
𝑀𝐴
𝜂 = × 100
𝑉𝑅

= 2.4 × 100
3
= 80%

Question 6
(a) (i) What is the principle of method of mixtures? [3]
(ii) What is the other name given to it?
(iii) Name the law on which the principle is based.
(b) Some ice is heated at a constant rate, and its temperature is recorded after every few [3]
seconds, till steam is formed at 100oC. Draw a temperature time graph to represent the
change. Label the two phase changes in your graph.
(c) A copper vessel of mass 100 g contains 150 g of water at 50°C. How much ice is needed [4]
to cool it to 5°C?
Given : Specific heat capacity of copper = 0.4 J g-1 ⁰C-1
Specific heat capacity of water = 4.2 J g-1 ⁰C-1
Specific latent heat of fusion of ice = 336 J g-1

Comments of Examiners
a) Most candidates did not mention the boundary Suggestions for teachers
statement such as no heat is lost to the surrounding or  Insist students on learning the laws
in an insulated system in the principal of mixtures. with limitations. Any law is obeyed
No mistakes were observed in the ii and iii part of under certain conditions.
this question.  For drawing graphs the axis should
b) Following errors were observed: be labelled. The graph line should be
 Axis were not labelled and boiling and melting drawn pertaining to the information
temperatures were not marked. given.
 Students should be made clear that in
 Graph line was drawn beyond 1000C and below
00C. these type of sums, equation depends
on how many substances are
 Phase change was not labelled.
involved in heat exchange. What is
c) While writing the equation many candidates did not
their temperature before exchange of
consider increase in temperature of melted ice to
heat and after the exchange of heat
50C. It was also observed that candidates were
and how many of them change their
unable solving numericals involving mixtures with
state. Adequate practice must be
change of state. It was also observed that they did
allotted for a better understanding of
not pay attention in writing the units correctly. For
this type of numerical.
eg many places gram was written as gm instead of g.

145
MARKING SCHEME
Question 6.
(a) (i) According to principle of mixtures Heat lost by a hot body is always equal to the heat gained
by a cold body provided no heat is lost to the surroundings
(ii) Principle of Calorimetry
(iii) Law of conservation of energy
(b)
Vaporization
100oC

temp

0oC Melting time


Correct graph (Axes and Shape)
Melting marked
Vaporization marked
(c) m c θ + m c θ = m I L + mi c θ
100 x 0.4 x 45 + 150 x 4.2 x 45 = mi ( 336 + 4.2 x 5 )
mi = 84.45 g

Question 7
(a) (i) Write a relationship between angle of incidence and angle of refraction for a given [3]
pair of media.
(ii) When a ray of light enters from one medium to another having different optical
densities it bends. Why does this phenomenon occur?
(iii) Write one condition where it does not bend when entering a medium of different
optical density.
(b) A lens produces a virtual image between the object and the lens. [3]
(i) Name the lens.
(ii) Draw a ray diagram to show the formation of this image.
(c) What do you understand by the term ‘Scattering of light’? Which colour of white light is [4]
scattered the least and why?

146
Comments of Examiners
a) In part (i) majority of candidates wrote is as ratio
Suggestions for teachers
of i to r instead of sin i to sin r. Some wrote i>r  Snell’s law should be clearly
when light enters from rarer to denser medium and explained with an emphasis on
i<r when light enters from denser to rarer medium. guiding students to interpret the
In part (ii) instead of writing reason for refraction question correctly and accordingly
candidates wrote due to refraction. This clearly write the correct answer with the
showed that they failed to understand the question. conditions attached.
In part (iii) instead of writing i = 00 some wrote 900
 While practicing diagram stress on
which was a careless error.
the following points.
b) Most candidates wrote the lens correctly as
 Arrows to be drawn on the ray
concave but some incorrectly wrote convex.
before and after the refraction.
Arrows were missing on the rays, image was not
dotted, apparent ray was not shown to pass through  Real image should be drawn by
the focus. These errors occurred due to lack of continuous line but virtual should
practice of drawing diagrams. be shown by dotted lines.
c) Most candidates were confused with the term  Diagram should be labelled
‘scattering’ in the definition of dispersion. neatly.
Majority however answered part (ii) and (iii)  Emphasize the difference between
correctly. However, some wrote violet due to least the definition of scattering and
wavelength. definition. Comparison can be done
on the basis of blue sky and rainbow.

MARKING SCHEME
Question 7.
(a) (i) sin (angle of incidence) / sin (angle of refraction) = Refractive Index.
(ii) The ray bends from its original path due to change in the speed.
(iii) The ray does not bend when incident normally at the surface of the second medium.
(b) (i) Concave lens.
(ii)

Any two rays correctly drawn


location of dotted image
(c) Absorption and reemission of light without the change in the wavelength.
Red
As its  is high.

147
Question 8
(a) (i) Name the waves used for echo depth sounding. [3]
(ii) Give one reason for their use for the above purpose.
(iii) Why are the waves mentioned by you not audible to us?

(b) (i) What is an echo? [3]


(ii) State two conditions for an echo to take place.
(c) (i) Name the phenomenon involved in tuning a radio set to a particular station. [4]
(ii) Define the phenomenon named by you in part (i) above.
(iii) What do you understand by loudness of sound?
(iv) In which units is the loudness of sound measured?

Comments of Examiners
a) Common errors observed were that candidates
wrote SONAR, RADAR, echo waves, radio Suggestions for teachers
waves and Ultraviolet in part (i). In part (ii)  To avoid confusion in the
answers were not related to least refraction, understanding of ultrasound and
narrow beam and high energy possessed. Some ultraviolet due to the common term
were confused to make decision whether to write “ultra” make the students aware
the use or property. In part (ii)i comparison with about this possible error and clearly
limit of audibility was not clear. It was just explain the difference between these
written beyond limit of audibility which does not two waves.
make it clear whether beyond 20 Hz or beyond  Bring out the difference between
20000 Hz. ordinary reflection of sound and echo
b) Candidates lost marks as they missed the to students. They should also be
keywords in writing the definition or a complete taught to explain that the minimum
definition. distance required for echo is the same
Keyword such as minimum for time and as the minimum time for reflection.
distance  Insist on writing the complete
was missing. Some wrote time and distance as definition and correct spellings of
two different points which actually means the scientific terms and units.
same.
c) Instead of resonance some candidates wrote
SONAR that also was misspelt. In the definition of resonance some made no mention about increase
in the amplitude and misspelt the unit of loudness.

148
MARKING SCHEME
Question 8.
(a) (i) Ultrasonic waves.
(ii) They can travel un-deviated through long distances.
(iii) Frequency of ultrasonic waves is above 20000Hz and audible range of frequency is 20Hz to
20000Hz.
(b) Clear and distinct sound heard after its reflection from a rigid surface is called an echo.
The reflecting surface should be present at least 16 to 18 m from the source of sound OR the
sound should be reflected back after 0.1 s.
The size of the reflecting surface should be greater than the wavelength of sound wave.
(c) (i) Resonance
(ii) Resonance is a particular case of forced vibration in which the frequency of forced vibrations is
equal to the natural frequency of the body and the body begins to vibrate with increased amplitude.
(iii) Loudness as magnitude of the auditory sensation
(iv) bel / dB / phon (any one)

Question 9

(a) (i) Which particles are responsible for current in conductors? [3]
(ii) To which wire of a cable in a power circuit should the metal case of a geyser be
connected?
(iii) To which wire should the fuse be connected?

(b) (i) Name the transformer used in the power transmitting station of a power plant. [3]
(ii) What type of current is transmitted from the power station?
(iii) At what voltage is this current available to our household?
(c) A battery of e.m.f. 12V and internal resistance 2 Ω is connected with two resistors A and [4]
B of resistance 4 Ω and 6 Ω respectively joined in series.

Find:
(i) Current in the circuit.
(ii) The terminal voltage of the cell.
(iii) The potential difference across 6Ω Resistor.
(iv) Electrical energy spent per minute in 4Ω Resistor.

149
Comments of Examiners
a) In part (i) majority of candidates wrote electrons Suggestions for teachers
but some incorrectly wrote protons or charge  Concept of flow of electrons under
carriers, cations, 𝛽 particle positive charge etc. In potential difference should be taught
part (ii) many wrote earth wire but some wrote to students. Video presentations may
neutral or live as they were confused. In part (iii) assist in clearing concepts. The
most candidates answered correctly. working of live, neutral and earth
b) Most candidates answered part (i) correctly as step wire should be clearly explained.
up transformer except for a few who incorrectly  Explain the difference between step
wrote step down transformer. In part (ii) many up and step down transformer and
candidates misinterpreted it as high or low rather make it clear how voltage and current
than A.C. or D.C. Some wrote A.C. of D.C (very are inversely proportional.
vague). Part (iii) while writing the voltage  The basic concept of series and
candidates wrote 20V or 330V or 290 V. Some parallel combination needs to be
forgot to mention the unit. addressed. Students should know
that in series P.D across each resistor
c) In part (i) , candidates appeared confused due to
is divided hence it is incorrect to use
the internal resistance. Some did not understand
e.m.f. to calculate P.D. across any
the meaning of terminal voltage and made vague
resistor. Units joule and kWh need to
calculations. To calculate P.D. across 6 Ω some
be explained clearly. Students should
used P.D. as 12 V. To calculate energy in joule
practice problems with internal
minutes were not converted to seconds. Some
resistance regularly.
divided by 60 and converted the time in hour and
wrote unit kWh without dividing by 1000.

MARKING SCHEME
Question 9.
(a) (i) Electrons
(ii) Earth wire
(iii) Live wire
(b) (i) Step up transformer
(ii) Alternating current
(iii) 220V
(c) (i) R = R 1 + R 2
= 4 Ω + 6 Ω = 10 Ω
𝑖 = 𝐸/𝑅 + 𝑟
= 12 / (10 + 2 )
= 1.0 A Current in the circuit = 1.0 A
(ii) 𝑇. 𝑉. = 𝐼𝑅

150
𝑇. 𝑉. = 1 × (6 + 4)
𝑇. 𝑉. = 10 𝑉
(iii) V = I R
=1x6
= 6.0 V
(iv) W = I 2 R t
= 1 x 1 x 4 x 60 = 240 J

Question 10

(a) Arrange α, β and γ rays in ascending order with respect to their [3]
(i) Penetrating power.
(ii) Ionising power.
(iii) Biological effect.
(b) (i) In a cathode ray tube what is the function of anode? [3]
(ii) State the energy conversion taking place in a cathode ray tube.
(iii) Write one use of cathode ray tube.
(c) (i) Represent the change in the nucleus of a radioactive element when a β particle is [4]
emitted.
(ii) What is the name given to elements with same mass number and different atomic
number?
(iii) Under which conditions does the nucleus of an atom tend to be radioactive?

151
Comments of Examiners
a) Candidates committed errors in the order of Suggestions for teachers
radiations. They were confused about ascending or - Students should read the question
descending and also in mathematical signs i.e. > or carefully as most errors occur due to
< used by them. carelessness. If the meaning of
b) Part (i) was answered correctly by most candidates ascending and descending is not
except for a few who wrote irrelevant answers that understood clearly then they should
reflected lack of understanding and preparation. write in complete sentences.
Many lost marks in part (ii) as they were not sure - Students need to understand when the
about energy change in CRT. Following question is asked about energy
incomplete answers were written electrical to heat, changes then visual pattern, visual
electrical to visual, electrical energy to visual terms does not mean energy. T.V.
pattern, electrical to visual signal, heat to light etc. means television which is a process
and one of the part is picture tube.
The expected answer was electrical to heat to light
energy. In part (iii) candidates did not stress on the stress on answers are to written in
words picture tube in spite of mentioning T.V. complete sentences.
- Make it clear to students that
c) Majority of candidates explained the reaction but represent means the answer is
failed to write an equation in spite of the question expected in the equation form. The
stating ‘Represent’ in part (i). In part (ii) some difference between isotopes and
candidates were not sure about the difference isobars should be explained with
between isotopes and isobars. In part (iii) examples. Students are to know that
candidates stated only one condition in spite of the what the nucleus becomes when the
question stating ‘which conditions’. nucleus has excess mass, energy and
when neutron proton ration becomes
more than 1.3 to 1.5 .

MARKING SCHEME
Question 10.
(a) (i) α<β<γ
(ii) γ<β<α
(iii) α<β<γ
(b) (i) Anode accelerates electrons and collimates into a fine beam.
(ii) Electric energy is converted into heat and then into light.
(iii) TV picture tube; check waveform of varying electric field
(any one use).
i) 1𝑛 → 1𝑃 + 0𝑒
(c) 0 (+)1 −1
ii) Isobars -------- 1
iii) When
-neutron proton ratio becomes more than 1.3 to 1.5
-nucleus has more/excess mass.
-nucleus has excess energy.
152
Topics / concepts found difficult:

 Students were confused in Q 4 a) and d) part as the nature of question was the same.
 Clarity was lacking in Q 6 a) i, ii, iii.
 It is also observed that in ray diagrams in spite of knowing diagrams students loose marks
as they don’t draw arrows on rays before and after refraction and don’t draw dotted line
for virtual intersection or image.
 Most of the students are confused between the concept of dispersion and scattering.
 In electricity the difference between P.D. and terminal voltage is not clear. In some cases
P.D. is misunderstood as only potential drop.
 Students were also confused about the working and the characteristics of transformers.
 Due to the lack of necessary information Q 2 e) was confusing.
 Q 5 a) ii, iii was asked beyond the depth of the topic which is covered in std. 10.
 In Q10 c) students were not aware about two conditions required for a nucleus to be
radioactive.
 Calorimetry numerical were found difficult.
 Q6 b) was beyond the scope of syllabus as it does not include latent heat of vaporization.
 Students were not comfortable in stating the factors affecting the frequency of stretched
string.

Suggestions to the students:

 Avoid writing answers which are simply a repetition of the question. Instead be specific about
the key word in that statement.
 Do not leave any topic for option. All topics are covered in section I which is compulsory.
 Avoid changing the order of sequence of questions and numbering system.
 Handwriting should be neat and legible.
 Learn the principles, laws and definitions accurately.
 Ray diagrams and the other diagrams need to be practiced periodically. While drawing them,
draw arrows on the rays before and after the refraction and virtual rays or image should be
drawn by dotted lines. No arrows to be drawn on virtual ray.
 While writing answers it is not only important to cover all points but also to present them in a
proper sequence.

153
 While solving a numerical it is advisable that the formula be written in the beginning. Essential
steps need to be shown and the final answer be expressed along with a proper unit.
 Avoid computation at the first step; let it be plain substitution as the marks are awarded for the
correct substitution.
 It is advisable to state the meaning of the symbols if the answer is given in terms of any
formula. Do not use any abbreviations which are not standard.
 The answer need to be given in SI units unless it is asked otherwise.
 In numerical as far as possible avoid mental calculation at the first stage, let it be direct
substitution.
 Advisable to present the final answer in the decimal form. Answer in fraction is treated
as incomplete calculation.
 It is advisable to solve previous year’s papers in writing.
 More emphasis should be given on writing rather than memorising.
 Units should be written without spelling errors.

154
ICSE SEMESTER 2 EXAMINATION
Class X Physics
Board Paper – 2021 (Set I)

Maximum Marks: 40
Time allowed: One and a half hours

Answers to this Paper must be written on the paper provided separately.

You will not be allowed to write during the first 10 minutes.


This time is to be spent reading the question paper.

The time given at the head of this Paper is the time allowed for writing the answers.

SECTION A
(ALL QUESTIONS ARE COMPULSORY.)

Question 1 [1]

The deviation produced by an equilateral prism does not depend on:

(a) the angle of incidence.

(b) the size of the prism.

(c) the material of the prism.

(d) the colour of light used.

77
Question 2 [1]

The refractive index of a diamond is 2.4. It means that:


1
(a) the speed of light in vacuum is equal to times the speed of light in diamond.
2.4

(b) the speed of light in the diamond is 2.4 times the speed of light in a vacuum.

(c) the speed of light in a vacuum is 2.4 times the speed of light in the diamond.

(d) the wavelength of light in diamond is 2.4 times the wavelength of light in vacuum.

Question 3 [1]

An object of height 10 cm is placed in front of a concave lens of focal length 20 cm at a distance 25 cm


from the lens. Is it possible to capture this image on a screen? Select a correct option from the following:

(a) Yes, as the image formed will be real.

(b) Yes, as the image formed will be erect.

(c) No, as the image formed will be virtual.

(d) No, as the image formed will be inverted.

Question 4 [1]

A ray of light IM is incident on a glass slab ABCD as shown in the figure below. The emergent ray for this
incident ray is:

(a) NQ

78
(b) NR

(c) NP

(d) NS

Question 5 [1]

The colour of white light which is deviated least by a prism is:

(a) green

(b) yellow

(c) red

(d) violet

Question 6 [1]

The wavelength range of visible light is:

(a) 40 nm to 80 nm

(b) 4000 nm to 8000 nm

(c) 4 nm to 8 nm

(d) 400 nm to 800 nm

Question 7

Observe the diagram which shows the path of an incident ray through an optical plane LL" of a lens.
The focal length of the lens is 20 cm.

79
(i) If an object is placed at a distance of 30 cm in front of this lens, then:

[1]

(a) the image will be virtual

(b) the image will be diminished and inverted.

(c) the image will be diminished.

(d) the image will be real and magnified.

(ii) This type of lens can be used: [1]

(a) to correct hypermetropia.

(b) to correct myopia.

(c) to diverge light.

(d) in the front door peepholes.

(iii) An object is placed in front of this lens at a distance of 60 cm. Then the image distance from the lens
with proper sign convention is: [1]

(a) +60 cm

(b) +30 cm

(c) -30 cm

(d) +15 cm

(iv) An object is placed in front of this lens at a distance of 60 cm. Then the magnification of the image is:
[1]

(a) 0.25

(b) 1.25

(c) -0.5

(d) 1

Question 8 [1]

The relation between CGS and S.I. unit of moment of force is:

(a) 1 Nm= 105 dyne cm

80
(b) 1 Nm= 105 dyne

(c) 1 Nm= 107 dyne cm

(a) 1 Nm= 105 N m

Question 9. [1]

A coolie raises a load upwards against the force of gravity then the work done by the load is:

(a) zero.

(b) positive work.

(c) negative work.

(d) none of these.

Question 10. [1]

The energy change during photosynthesis in plants is:

(a) heat to chemical.

(b) light to chemical.

(c) chemical to light.

(d) chemical to heat.

Question 11. [1]

The diagram below shows the balanced position of a metre scale.

Which one of the following diagrams shows the correct position of the scale when it is supported at the
centre?

81
(a)

(b)

(c)

(d)

Question 12.

A stone tied at the end of a string is whirled by hand in a horizontal circle with uniform speed.

(i) Name the force required for this circular motion:

[1]

(a) Centrifugal force

(b) Centripetal force

(c) Force of gravity

82
(d) Frictional force

(ii) What is the direction of the above-mentioned force? [1]

(a) Towards the centre of the circular path.

(b) Away from the centre of the circular path.

(c) Normal to the radius at a point where the body is present on the circular path.

(d) Direction of this force keeps on changing alternately towards and away from the centre.

Question 13

A body of mass 200 g falls freely from a height of 15 m. [g = 10 m s−2]

(i) When the body reaches 10 m above the ground, its potential energy will be: [1]

(a) 20000 J

(b) 10 J

(c) 10000 J

(d) 20 J

(ii) The gain in kinetic energy of the body when it reaches 10 m above the ground is: [1]

(a) 20 J

(b) 10 J

(c) 30 J

(d) 25 J

(iii) The total mechanical energy it will possess, when it is just about to strike the ground is:
[1]

(a) 30000 J

(b) 20000 J

(c) 30 J

(d) 20 J

(iv) The velocity in ms-1 with which the body will hit the ground is: [1]

(a) 30

(b) 10
83
(c) 10√3

(c) 10√2

Question 14 [1]

A woman draws water from a well using a fixed pulley. The mass of the bucket and the water together is 10
kg. The force applied by the woman is 200 N. The mechanical advantage is (g = 10 m/𝑠2):

(a) 2

(b) 20

(c) 0.05

(d) 0.5

Question 15 [1]

A single fixed pulley is used because:

(a) it changes the direction of applied effort conveniently.

(b) it multiplies speed.

(c) it multiplies effort.

(d) its efficiency is 100%.

Question 16. [1]

In the diagram shown below, the velocity ratio of the arrangement is:

84
(a) 1

(b) 2

(c) 3

(d) 0

Question 17 [1]

Which one of the following is the correct mathematical relation?

(a) Power = Force/ Velocity

(b) Power = Force x Acceleration

(c) Power Force/ Acceleration

(d) Power= Force x Velocity

Question 18 [1]

Select a correct option with respect to echo depth sounding:

(a) infrasonic waves are used

(b) the frequency of the waves used is between 20 Hz and 20,000 Hz.

(c) ultrasonic waves are used.

(d) supersonic waves are used.

85
Question 19 [1]

Which one of the following diagnostic methods use reflection of sound?

(a) CT scan

(b) Electrocardiogram

(c) Echo cardiogram

(d) MRI

Question 20.

A boy standing in front of a wall produces two whistles per second. He notices that the sound of his
whistling coincides with the echo. The echo is heard only once when whistling is stopped. Calculate the
distance between the boy and the wall. (The speed of sound in air = 320 m/s)

(i) The time in which the boy hears the echo is: [1]

(a) 1 s

(b) 0.5 s

(c) 1.5 s

(d) 2 s

(ii) The distance at which the boy is standing from the wall: [1]

(a) 160 m

(b) 240 m

(c) 320 m

(d) 80 m

(iii) If the speed of sound is increased by 16 ms-1 and the boy moves 4 m away from the wall then in how
much time will he hear the echo of the first whistle? [1]

(a) 0.525 s

(b) 0.5 s

(c) 0.48 s

(d) 0.3 s
86
(iv) In which of the following timings of reflection of the whistle, the echo cannot be
heard? [1]

(a) 0.05 s

(b) 0.12 s

(c) 0.2 s

(d) 0.11 s

Question 21 [1]

The ratio of velocities of light of wavelength 400 nm and 800 nm in a vacuum is:

(a) 1:1

(b) 1:2

(c) 2:1

(d) 1:3

Question 22 [1]

1 joule = erg

(a) 109

(b) 107

(c) 105

(d) 106

Question 23

A light body A and a heavy body B have the same momentum.

(i) Choose a correct statement from the given options. [1]

(a) kinetic energy of body A and body B will be the same.

(b) kinetic energy of body A is greater than kinetic energy of body B.

87
(c) kinetic energy of body B is greater than kinetic energy of body A.

(d) unless we know the velocity, we cannot find which body has greater kinetic energy.

(ii) If the ratio of kinetic energies of A and B is 5 : 2 then which one of the following gives the mass ratio of
the bodies respectively? [1]

(a) 5 : 2

(b) 25 : 4

(c) 2 : 5

(d) 4 : 24

Question 24

The diagram below shows a ray of light traveling from air into a glass material as shown below. Answer the
questions that follow:

(i) The angle of incidence at the surface AB is: [1]

(a) 430

(b) 470

(c) 900

(d) 00

(ii) Select a correct statement from the following. [1]

(a) The speed of light at the curved surface AD does not change while entering the block.

88
(b) The ray at the surface AD is not travelling along the radius of the curved part

(c) The ray at the surface AD is travelling along the radius of the curved part.

(d) Light never refracts when it enters a curved surface.

(iii) The angle of incidence on the surface BC is: [1]

(a) 430

(b) 470

(c) 900

(d) 00

(iv) The critical angle of this material of glass: [1]

(a) 470

(b) 430

(c) 420

(d) 450

Question 25.

The diagram below shows the path of light passing through a right-angled prism of critical angle 𝟒𝟐𝟎.

(i) The angle C of the prism is: [1]

(a) 450

(b) 600
89
(c) 900

(d) 300

(ii) Which one of the following diagrams shows the correct path of this ray till it emerges out of the prism?
[1]

(a)

(b)

(c)

(d)

90
Solutions

Solution 1 Correct option – b) size of prism The deviation produced by an equilateral prism does not
depend on size of prism.

Solution 2 Correct option – c) the speed of light in vacuum is 2.4 times the speed of light in the diamond.

Refractive index,
𝐶
𝜇 = 𝜇𝑑𝑖𝑎𝑚𝑜𝑛𝑑 = = 2.4
𝑑𝑖𝑎𝑚𝑜𝑛𝑑 𝜇𝑎𝑖𝑟 𝑉𝑑𝑖𝑎𝑚𝑜𝑛𝑑

where, c velocity of light in air/vacuum Thus,

𝑐 = 2.4 × 𝑉diamond
Thus, the speed of light in a vacuum is 2.4 times the speed of light in the diamond.

Solution 3 Correct option – c) No, as the image formed will be virtual.

As the lens used is concave lens, the image formed will be virtual and erect and such images cannot be
obtained on the screen.

Solution 4 Correct option – a) NQ

The light ray MS is extension of incident ray IM.

When light ray is coming out from the denser medium into rarer medium the emergent ray for glass slab is
parallel to that of incident ray and it bends towards the normal. The only ray which is satisfying both the
condition is ray NQ.

Solution 5 Correct option – c) red

The colour of white light which is deviated least by a prism is red.

Solution 6 Correct option – d) 400 nm to 800 nm

The wavelength range of visible light is 400 nm to 800 nm.

91
Solution 7

i) Correct option – d) the image will be real and magnified It is given that after refracting the rays of light
are converging at point to right side of lens. The object is placed at 30 cm and focal length of lens is 20 cm.
This means as the rays are converging the lens is convex lens and object is placed between 2F and F. Thus,
the image formed will be real and magnified.

ii) Correct option – a) to correct hypermetropia. The lens used is convex lens. The explanation for the same
is given in 7 (i). Convex lens is used to correct hypermetropia.

iii) Correct option – b) +30 cm When the object is placed at 60 cm to left side of convex lens of focal
length 20 cm, it means that the object is placed beyond 2F. And when the object is placed beyond 2F of
convex lens the image formed will be between F and 2F (i.e., between + 20 cm and + 40 cm) to the right
side. Thus, the sign according to New Cartesian sign convention will be positive for the image distance. The
only valid option satisfying all these conditions is + 30 cm.

iv) Correct option – c) – 0.5

As the image formed in this case will be real and inverted the sign to the value of the height of image
should be negative according to New Cartesian sign convention. And as the object is placed beyond 2F the
image formed will be diminished. This means the image formed will be less than 1. The value satisfying
both the conditions is – 0.5.

Solution 8 Correct option – c) 1 Nm = 107 dyne cm

Moment of force = Force x perpendicular distance from line of action. Thus, SI unit of moment of force is
Nm and SI unit is dyne-cm.

1 N =105 dyne

1 m = 100 cm

Thus, 1 Nm = 107 dyne-cm

Solution 9 Correct option – c) negative work

The coolie has raised the load in upward direction i.e., displacement is in opposite direction to that of force
of gravity and when this is the case the work done by load is negative. Thus, a negative work is done.

Solution 10 Correct option – b) light to chemical

The energy change during photosynthesis in plants is light energy to chemical energy.

92
Solution 11 Correct option – a)

As given in the question we can see that when the fulcrum is at 40 cm the left arm and right arm both are in
balanced position. So, when the fulcrum is moved to the centre of scale which is 50 cm (i.e., to the right
side of the scale) the left side will weigh more than right and thus, left side will move in downward
direction. Thus, correct option is a)

Solution 12

i) Correct option – b) centripetal force

A stone tied at end of a string is whirled by hand in a horizontal circle with uniform speed. The force
required for this circular motion is centripetal force.

ii) Correct option – a) towards the centre of the circular path

In the given case, the direction of force is towards the centre of the circular path.

Solution 13

i) Correct option – d) 20 J

The potential energy is given as,

P.E. = mgh

M = 200 g = 0.2 kg

g = 10 m/s2 (given), h = 10 m

P.E. = 0.2 × 10 × 10 = 20 J

ii) Correct option – b) 10 J

The potential energy at the highest point in this case, 15 m is

P.E. = mgh = 0.2 x 15 x 10 = 30 J

Potential energy at height 15 m is maximum and the kinetic energy is zero.

When body reaches 10 m, the potential energy become 20 J

93
According to law of conservation of energy, the kinetic energy at 10 m height will be P.E. at height 15 m –
P.E. at height 10 m = Gain in K.E. = 30 J – 20 J = 10 J

Thus, gain in K.E. until it reaches 10 m above ground is = 10 J

iii) Correct option – c) 30 J

According to law of conservation of energy the total energy at any point of its journey during this freefall
will remain same.

Thus, total energy of body when it is about to strike ground is 30 J.

iv) Correct option – c) 10√3

When the body will hit the ground the kinetic energy of this body will be maximum. This means Kinetic
energy at ground will be 30 J.

We know,

K.E. = ½ 𝑚𝑣2

30 = 1 x 0.2 x 𝑣2
2

𝑣2 = 30 𝑥 2 = 300
0.2

V = √300 = √3 𝑥 100 = √3 𝑥 25 𝑥 4 = 10√3

Thus, velocity of the body with which it will hit the ground is 10√3 m/s.

Solution 14 Correct option – d) 0.5

Load = 10 kgf = 100 N

Effort = 200 N

We know,

Mechanical advantage = Load/ Effort = 100/200 = 0.5

Solution 15 Correct option – a) it changes the direction of applied effort conveniently A single fixed
pulley is used because it changes the direction of applied effort conveniently.

94
Solution 16 Correct option – b) 2

The velocity ratio of the arrangements of the pulley in which one is fixed and another one is movable is 2.

Solution 17 Correct option – d) Power = Force × Velocity

Power = Force × Velocity is the correct mathematical relation.

Solution 18 Correct option – c) ultrasonic waves are used.

In echo depth sounding, ultrasonic waves are used.

Solution 19 Correct option – c) Echo cardiogram

The diagnostic method which uses reflection of sound is echo cardiogram.

Solution 20

i) Correct option – b) 0.5 s

The boy blows two whistles per second. That means time taken for one whistle is 0.5 s. The sound of
whistling coincide with the echo and echo is heard only when whistling is stopped. This means the time in
which the boy hears echo will be 0.5 s

ii) Correct option – d) 80 m

We know,

d = (v x t)/ 2 = (320 x 0.5)/2 = 160/2 = 80 m

iii) Correct option – b) 0.5 s

When the speed of sound is increased by 16 m/s. The new speed becomes 336 m/s The boy moves 4 m
away from the wall i.e., now he will be standing 84 m away from the wall.

We know,

d = (v x t)/2

84 = (320 x t)/2

84 = (336 x t)/ 2

t = (84 x 2)/336 = 0.5 s


95
iv) Correct option – a) 0.05 s

For the echo to be heard the minimum time must be 1/10th of the second i.e., 0.1 of second. So, the echo
cannot be heard at 0.05 s.

Solution 21 Correct option – a) 1 : 1

When the light is travelling in a vacuum, the velocity of light of any wavelength is same. Thus, ratio of
velocities of light of wavelength 400nm and 800 nm in a vacuum is 1:1.

Solution 22 Correct option – b) 107

1 J =107 erg

Solution 23

i) Correct answer – b) Kinetic energy of body A is greater than kinetic energy of body B

Given that,

Momentum of body A, 𝑃𝐴=Momentum of body B, 𝑃𝐵Mass of body A, 𝑚𝐴 < Mass of body B, 𝑚𝐵 Hence the
ratio of kinetic energy of body A and B are

𝐾. 𝐸𝐴 5
=
𝐾. 𝐸𝐵 2

Now we know the relation between kinetic energy and mass of body and it can be expressed as

Solution 24

i) Correct option – b) 47°

From the given figure we can see that angle of incidence i.e., the angle between incident ray and normal is
47° along surface AB as we can see below. Hence the angle of incidence at surface AB is 47° (i.e., AB =90°
- 43°).

96
ii) Correct option –c) The ray at the surface AD is travelling along the radius of curved part. For the given
case we know that light is incident on a curved surface of glass material.

Hence the ray will be travelling along the radius of curvature since light incident perpendicular to curved
surface will always travel towards its centre as shown below.

iii) Correct option – a) 43°

From the given figure we can see that angle of incidence along surface BC will be 43° as we can see below.

97
Alternate method:

Here for triangle FOE the angle of incidence i will be given as we can see below. ∠𝑖 = 180−47−90=43°

iv) Correct option – a) 43° As we can see from the given figure, when angle of incidence at surface BC is
43° the light ray becomes parallel to surface BC. Hence for the given case the angle of incidence is 43°.

Solution 25

i) Correct option – d) 30°

From the given figure we can see that angle between incident ray and normal
across surface AC is 60°. Using this we can predict that angle A will be 30° and
angle C will be 60° by using triangle sum theorem (i.e., ∠A+ ∠B+ ∠C=180°).

Hence the angle C of given prism will be 3

ii) Correct option – b)

98
0°.

From the given data we know that critical angle of prism is 42° and angle of incidence is 60°.

Thus, from this we can conclude that option c is incorrect since incident angle is greater than critical angle
(i.e., ∠i > ∠ic) which means light will undergo total internal reflection. And from this given figure we can
conclude that only option b is correct since for option a incident angle is not equal to reflected ray and for
option d light ray after refracting from surface BC will bend away from normal not toward normal (∵ 𝜇𝑔 >
𝜇𝑎𝑖𝑟).

99

You might also like